JUMC Pharm V

अब Quizwiz के साथ अपने होमवर्क और परीक्षाओं को एस करें!

Which of the following is a common direct or reflex effect of nitroglycerin? (A) Decreased heart rate (B) Decreased venous capacitance (C) Increased afterload (D) Increased cardiac force (E) Increased diastolic myocardial fiber tension

Nitroglycerin increases heart rate and venous capacitance and decreases afterload and diastolic fiber tension. It increases cardiac contractile force because the decrease in blood pressure evokes a compensatory increase in sympathetic discharge. The answer is D.

Which one of the following is the most common side effect of antihyperlipidemic drug therapy? A. Elevated blood pressure. B. Gastrointestinal disturbance. C. Neurologic problems. D. Heart palpitations. E. Migraine headaches.

B. Gastrointestinal disturbance.

Consumption of alcohol is associated with which of the following changes in serum lipid concentrations? (A) Decreased chylomicrons (B) Decreased HDL cholesterol (C) Decreased VLDL cholesterol (D) Increased LDL cholesterol (E) Increased triglyceride

Chronic ethanol ingestion can increase serum concentrations of VLDL and triglycerides. This is one of the factors that places patients with alcoholism at risk of pancreatitis. Chronic ethanol ingestion also has the possibly beneficial effect of raising, not decreasing, serum HDL concentrations. The answer is E.

All of the following are adverse effects of amiodarone except: A. Cinchonism. B. Hypothyroidism. C. Hyperthyroidism. D. Pulmonary fibrosis. E. Blue skin discoloration.

Correct answer = A. Cinchonism is a constellation of symptoms (blurred vision, tinnitus, headache, psychosis) that is known to occur with quinidine. All other options are adverse effects with amiodarone that require close monitoring.

A 3-year-old healthy female ingested one of her mother's 1 mg alprazolam tablets 45 minutes ago. The child presented to the emergency department with CNS depression but a normal heart rate and blood pressure. Her bedside glucose check is also normal. Which of the following antidotes might be helpful? A. Flumazenil. B. Naloxone. C. Physostigmine. D. Atropine. E. Fomepizole.

Correct answer = A. Flumazenil is a competitive benzodiazepine antagonist that reverses the CNS depression from benzodiazepines such as alprazolam. After flumazenil administration, resedation usually occurs, since the duration of the benzodiazepine is longer than that of the flumazenil. Naloxone reverses the effects from opioids and clonidine, not benzodiazepines. Physostigmine is the antidote for anticholinergic toxicity, and atropine is an anticholinergic agent. Fomepizole is the antidote for methanol or ethylene glycol toxicity.

A 40-year-old male presents to the emergency department with a complaint of abdominal pain. The patient appears intoxicated, but an ethanol level returns as negative and his basic metabolic panel is unremarkable. Which of these substances did he probably ingest? A. Isopropyl alcohol. B. Methanol. C. Ethylene glycol. D. Ethanol. E. Organophosphates

Correct answer = A. Isopropyl alcohol produces twice as much CNS depression as ethanol and is known to cause GI distress. Isopropyl alcohol is metabolized to acetone, so a metabolic acidosis does not result (which is in contrast to the acidosis generated by methanol and ethylene glycol). The ethanol level was negative, eliminating ethanol as an ingestion. Organophosphate toxicity yields nicotinic and muscarinic effects, which are not described in the history.

Questions 5 and 6. A patient was treated for a bacterial infection with a penicillin. Within a few minutes of the antibiotic injection, he developed severe bronchoconstriction, laryngeal edema, and hypotension. Because of the rapid administration of epinephrine, the patient survived. Unfortunately, a year later he was treated with an antipsychotic drug and developed agranulocytosis. 5. Which type of immunologic process was triggered by the penicillin injection? (A) An autoimmune syndrome (B) A cell-mediated reaction (C) A type II drug allergy (D) Mediated by IgE (E) Serum sickness 6. Which type of immunologic process was triggered by the antipsychotic drug? (A) A type III drug reaction (B) A type IV drug reaction (C) Delayed-type hypersensitivity (D) Mediated by IgG or IgM antibodies (E) Stevens-Johnson syndrome

5. The patient experienced an anaphylactic response to the penicillin. This is a type I (immediate) drug reaction, mediated by IgE antibodies. The answer is D. 6. Agranulocytosis (and systemic lupus erythematosus) are autoimmune syndromes that can be drug-induced. They are type II reactions involving IgM and IgG antibodies that bind to circulating blood cells. The patient was probably treated with clozapine for his psychosis (see clozapine toxicity, Chapter 29). The answer is D.

A 45-year-old woman with hyperlipidemia and frequent migraine headaches develops angina of effort. Which of the following is relatively contraindicated because of her migraines? (A) Amlodipine (B) Diltiazem (C) Metoprolol (D) Nitroglycerin (E) Verapamil

Acute migraine headache is associated with vasodilation of meningeal arteries. Of the drugs listed, only nitroglycerin is commonly associated with headache. In fact, calcium channel blockers and β blockers have been used with some success as prophylaxis for migraine. The answer is D.

Which one of the following drugs is associated with clinically useful or physiologically important positive inotropic effect? (A) Captopril (B) Dobutamine (C) Enalapril (D) Losartan (E) Nesiritide

Although they are extremely useful in heart failure, ACE inhibitors (eg, captopril, enalapril), and angiotensin receptor blockers (ARBs, eg, losartan) have no positive inotropic effect on the heart. Nesiritide is a vasodilator with diuretic effects and renal toxicity. Dobutamine is a β1-selective adrenoceptor agonist. The answer is B.

A 57-year-old man is being treated for an atrial arrhythmia. He complains of dry mouth, blurred vision, and urinary hesitancy. Which antiarrhythmic drug is he mostly like taking? A. Metoprolol. B. Disopyramide. C. Dronedarone. D. Sotalol.

Correct answer = B. The clustered symptoms of dry mouth, blurred vision, and urinary hesitancy are characteristic of anticholinergic adverse effects which are caused by class IA agents (in this case, disopyramide). The other drugs do not cause anticholinergic effects.

Which of the following medications would be safe to use in a patient taking ranolazine? A. Carbamazepine. B. Clarithromycin. C. Enalapril. D. Quetiapine.

Correct answer = C. All other medications should be avoided due to potential drug-drug interactions.

Suppression of arrhythmias resulting from a reentry focus is most likely to occur if the drug: A. Has vagomimetic effects on the AV node. B. Is a β-blocker. C. Converts a unidirectional block to a bidirectional block. D. Slows conduction through the atria. E. Has atropine-like effects on the AV node

Correct answer = C. Current theory holds that a reentrant arrhythmia is caused by damaged heart muscle, so that conduction is slowed through the damaged area in only one direction. A drug that prevents conduction in either direction through the damaged area interrupts the reentrant arrhythmia. Class I antiarrhythmics, such as lidocaine, are capable of producing bidirectional block. The other choices do not have any direct effects on the direction of blockade of conduction through damaged cardiac muscle.

A patient whose angina was previously well controlled with once-daily isosorbide mononitrate states that recently he has been taking isosorbide mononitrate twice a day to control angina symptoms that are occurring more frequently during early morning hours. Which of the following is the best option for this patient? A. Continue once-daily administration of isosorbide mononitrate but advise the patient to take this medication in the evening. B. Advise continuation of isosorbide mononitrate twice daily for full 24-hour coverage of anginal symptoms. C. Switch to isosorbide dinitrate, as this has a longer duration of action than the mononitrate. D. Switch to nitroglycerin patch for consistent drug delivery and advise him to wear the patch around the clock.

Correct answer = A. It is important to maintain a nitrate-free period to prevent the development of tolerance to nitrate therapy. The mononitrate formulation has the longer halflife. The nitroglycerin patch should be taken off for 10 to 12 hours daily to allow for nitrate-free interval.

Cyclosporine is effective in organ transplantation. Which of the following most accurately describes the immunosuppressant action of cyclosporine? (A) Activation of NK cells (B) Blockade of tissue responses to inflammatory mediators (C) Increased catabolism of IgG antibodies (D) Inhibition of the gene transcription of interleukins (E) Interference with MHC II-peptide activation of T cells

Cyclosporine inhibits calcineurin, a serine phosphatase that is needed for activation of T-cell-specific transcription factors such as NF-AT. Gene transcription of IL-2, IL-3, and interferon-γ is inhibited. The answer is D.

A 24-year-old man was employed in the supply department of a company that manufactures semiconductors. After an accident at the plant, he presented with nausea and vomiting, headache, hypotension, and shivering. Laboratory analyses showed hemoglobinuria and a plasma free hemoglobin level greater than 1.4 g/dL. This young man was probably exposed to (A) Arsine (B) Inorganic arsenic (C) Mercury vapor (D) Methylmercury (E) Tetraethyl lead

From the signs and symptoms alone, a diagnosis of arsine gas poisoning cannot be made. However, clues to the cause of poisoning are often provided by a patient's occupation. The laboratory reports suggest marked hemolysis. Arsine gas binds to hemoglobin and decreases erythrocyte glutathione levels, causing membrane fragility and resulting hemolysis. The answer is A.

PJ is a 4.5-year-old boy. At his checkup, the pediatrician notices cutaneous xanthomas and orders a lipid panel. Repeated measures confirm that the patient's serum cholesterol levels are high (936 mg/dL). Further testing confirms a diagnosis of homozygous familial hypercholesterolemia. Which of the following interventions will be least effective in this patient? (A) Atorvastatin (B) Ezetimibe (C) Lomitapide (D) Mipomersen (E) Niacin

Homozygous familial hypercholesterolemia is caused by mutations leading to dysfunctional LDL receptors incapable of taking up LDL from the bloodstream. Options B-E would have a cholesterol-lowering effect. Lomitapide and mipomersen are specifically indicated for patients with familial hypercholesterolemia. Reductase inhibitors such as atorvastatin rely on functional LDL receptors to achieve a LDL-lowering effect and thus will not work in patients with homozygous familial hypercholesterolemia. The answer is A.

She is excited about not having to come in for blood tests but wonders if there is a test, just in case the doctors need to know. Which of the following tests would provide accurate information about the coagulation status of a patient taking apixaban? (A) aPTT (B) Factor X test (C) INR (D) PT test

INR (measured as PT test) reflects changes due to warfarin and to some extent the thrombin inhibitors. Factor X inhibition is not reliably measured by the aPTT (used for unfractionated heparin) or PT test. The answer is B

Which of the following is the best-documented mechanism of beneficial action of cardiac glycosides? (A) A decrease in calcium uptake by the sarcoplasmic reticulum (B) An increase in ATP synthesis (C) A modification of the actin molecule (D) An increase in systolic cytoplasmic calcium levels (E) A block of cardiac β adrenoceptors

Digitalis does not decrease calcium uptake by the sarcoplasmic reticulum or increase ATP synthesis; it does not modify actin. Cardiac adrenoceptors are not affected. The most accurate description of digitalis's mechanism in this list is that it increases systolic cytoplasmic calcium indirectly by inhibiting Na+/K+ ATPase and altering Na/Ca exchange. The answer is D.

A patient develops severe thrombocytopenia in response to treatment with unfractionated heparin and still requires parenteral anticoagulation. The patient is most likely to be treated with which of the following? (A) Abciximab (B) Bivalirudin (C) Tirofiban (D) Plasminogen (E) Vitamin K1

Direct thrombin inhibitors such as bivalirudin and argatroban provide parenteral anticoagulation similar to that achieved with heparin, but the direct thrombin inhibitors do not induce formation of antiplatelet antibodies. The answer is B.

A small child is brought to a hospital emergency department suffering from severe gastrointestinal distress and abdominal colic. If this patient has severe acute lead poisoning with signs and symptoms of encephalopathy, treatment should be instituted immediately with (A) Acetylcysteine (B) Deferoxamine (C) EDTA (D) Penicillamine (E) Succimer

Encephalopathy in severe lead poisoning is a medical emergency. Of the drugs listed, intravenous EDTA is the most effective chelating agent. Oral succimer is used in children with mild to moderate lead poisoning and may be initiated 4-5 d after the parenteral use of EDTA or dimercaprol in severe poisoning. The answer is C.

Which of the following is an immune modulator that increases phagocytosis by macrophages in patients with chronic granulomatous disease? (A) Aldesleukin (B) Interferon-γ (C) Lymphocyte immune globulin (D) Prednisone (E) Trastuzumab

Interferon-γ is approved for use in chronic granulomatous disease, a condition that results from phagocyte deficiency. The agent markedly reduces the frequency of recurrent infections. The answer is B.

Recombinant interleukin-2 has proved useful in the treatment of which of the following diseases? (A) Graft-versus-host disease in patients with hematopoietic stem cell transplantation (B) Psoriasis (C) Renal cell carcinoma (D) Rheumatoid arthritis (E) Superficial bladder carcinoma

Interleukin-2 is a cytokine that stimulates T-cell proliferation and activates Th1, NK, and LAK cells. It has shown efficacy in renal cell carcinoma and malignant melanoma, 2 cancers that respond poorly to conventional cytotoxic anticancer drugs. The answer is C.

A 32-year-old woman with hypertension wishes to become pregnant. Her physician informs her that she will have to switch to another antihypertensive drug. Which of the following drugs is absolutely contraindicated in pregnancy? (A) Atenolol (B) Losartan (C) Methyldopa (D) Nifedipine (E) Propranolol

Methyldopa is often recommended in pregnant patients because it has a good safety record. Calcium channel blockers (choice D) and β blockers (choices A and E) are not contraindicated. In contrast, ACE inhibitors and ARBs (choice B) have been shown to be teratogenic. The answer is B.

An employee of a company engaged in clearing vegetation from county roadsides accidentally ingested a small quantity of an herbicidal solution that contained paraquat. Within 2 h, he was admitted to the emergency department of a nearby hospital. Which of the following best describes his probable signs and symptoms in the emergency department? (A) Diarrhea, vomiting, sweating, and profound skeletal muscle weakness (B) Dizziness, nausea, agitation, and hyperreflexia (C) Dyspnea, pulmonary dysfunction, and elevated body temperature (D) Gastrointestinal irritation with hematemesis and bloody stools (E) Hypotension, tachycardia, and respiratory impairment

Paraquat is highly corrosive to the gastrointestinal tract. Oral ingestion of the herbicide leads to marked gastrointestinal irritation, hematemesis, and usually blood in the stools. Signs of pulmonary impairment do not appear for several days and are usually progressive, resulting in severe pulmonary fibrosis and, often, death. The answer is D.

A patient who has been taking digoxin for several years for atrial fibrillation and chronic heart failure is about to receive atropine for another condition. A common effect of digoxin (at therapeutic blood levels) that can be almost entirely blocked by atropine is (A) Decreased appetite (B) Headaches (C) Increased atrial contractility (D) Increased PR interval on ECG (E) Tachycardia

The parasympathomimetic effects of digitalis can be blocked by muscarinic blockers such as atropine. The only parasympathomimetic effect in the list provided is increased PR interval, a manifestation of slowed AV conduction. The answer is D.

A patient is brought to the emergency department having taken an overdose (unknown quantity) of a sustained-release preparation of theophylline by oral administration 2 h previously. He has marked gastrointestinal distress with vomiting, is agitated, and exhibits hyperreflexia and hypotension. A short-acting antidote that can reduce this patient's tachycardia is (A) Acetylcysteine (B) Deferoxamine (C) Esmolol (D) Fomepizole (E) Pralidoxime

The short-acting β blocker esmolol helps reverse the tachycardia and possibly the vasodilation associated with an overdose of theophylline. The answer is C.

You are stuck in traffic in New York City in summer for 3 or 4 h and you begin to get a headache, a feeling of tightness in the temporal region, and an increased pulse rate. What is the antidote based on the most likely cause of these effects? (A) Activated charcoal (B) Atropine (C) Fomepizole (D) Oxygen (E) Pralidoxime

The symptoms described are those of carbon monoxide inhalation. Oxygen is the antidote. The answer is D. Note atropine and pralidoxime are used in insecticide poisoning with acetylcholinesterase inhibitors, and fomepizole is used in methanol and ethylene glycol poisoning.

A young engineer involved in the smelting process of cobalt and gold presented with severe GI discomfort, rice water stools, and a sweet garlicky breath. Acute inorganic arsenic poisoning was diagnosed. Which of the following drugs should be included in the management of this patient? (A) Deferoxamine (B) Dimercaprol (C) EDTA (D) Penicillamine (E) Succimer

The treatment of choice in acute arsenic poisoning is intramuscular dimercaprol. Although succimer is less toxic, it is only available in an oral formulation, and its absorption may be impaired by the severe gastroenteritis that occurs in acute arsenic poisoning. The answer is B.

Which of the following is very short-acting and acts by releasing nitric oxide? (A) Atenolol (B) Captopril (C) Diltiazem (D) Fenoldopam (E) Hydrochlorothiazide (F) Losartan (G) Minoxidil (H) Nitroprusside (I) Prazosin

The two agents in this list that act via a nitric oxide mechanism are hydralazine and nitroprusside (see Table 11-2). However, hydralazine has a duration of action of hours, whereas nitroprusside acts for seconds to minutes and must be given by intravenous infusion. The answer is H.

A 2-year-old child was brought to the emergency department 1 h after ingestion of tablets he had managed to obtain from a bottle on top of the refrigerator. His symptoms included marked gastrointestinal distress, vomiting (with hematemesis), and epigastric pain. Metabolic acidosis and leukocytosis were also present. This patient is most likely to have ingested tablets containing (A) Acetaminophen (B) Aspirin (C) Diphenhydramine (D) Iron (E) Vitamin C

This question emphasizes that the ingestion of iron tablets is a relatively common cause of accidental poisoning in young children. The signs and symptoms described usually occur in the first 6 h after ingestion. In a child whose body weight is 22 lb, the ingestion of 600 mg can cause severe, perhaps lethal, toxicity. The answer is D.

Certain drugs can cause severe hypotension when combined with nitrates. Which of the following interacts with nitroglycerin by inhibiting the metabolism of cGMP? (A) Atenolol (B) Hydralazine (C) Isosorbide mononitrate (D) Nifedipine (E) Ranolazine (F) Sildenafil (G) Terbutaline

Sildenafil inhibits phosphodiesterase 5, an enzyme that inactivates cGMP. The nitrates (via nitric oxide) increase the synthesis of cGMP. This combination is synergistic. The answer is F.

The light brownish color of smog often apparent in a major metropolitan area on a hot summer day is mainly due to (A) Carbon monoxide (B) Hydrocarbons (C) Ozone (D) Nitrogen dioxide (E) Sulfur dioxide

Smog color is derived in part from suspended particulate matter. When smog is light brown, the color derives from nitrogen oxides. All of the other air pollutants listed are colorless. The answer is D.

After being counseled about lifestyle and dietary changes, the patient was started on atorvastatin. During his treatment with atorvastatin, it is important to routinely monitor serum concentrations of which of the following? (A) Blood urea nitrogen (B) Alanine and aspartate aminotransferase (C) Platelets (D) Red blood cells (E) Uric acid

The 2 primary adverse effects of the HMG-CoA reductase inhibitors are hepatotoxicity and myopathy. Patients taking these drugs should have liver function tests performed before starting therapy, and at regular intervals as needed during therapy. Serum concentrations of alanine and aspartate aminotransferase are used as markers of hepatocellular toxicity. The answer is B.

Which of the following is a widely used drug that suppresses cellular immunity, inhibits prostaglandin and leukotriene synthesis, and increases the catabolism of IgG antibodies? (A) Cyclophosphamide (B) Cyclosporine (C) Infliximab (D) Mycophenolate mofetil (E) Prednisone

The corticosteroid prednisone is used extensively as an immunosuppressant in autoimmune diseases and organ transplantation. Glucocorticoids have multiple actions, including those described. The answer is E.

A 55-year-old man is admitted to the emergency department and is found to have an abnormal ECG. Overdose of an antiarrhythmic drug is considered. Which of the following drugs is correctly paired with its ECG effects? (A) Quinidine: Increased PR and decreased QT intervals (B) Flecainide: Increased PR, QRS, and QT intervals (C) Verapamil: Increased PR interval (D) Lidocaine: Decreased QRS and PR interval (E) Metoprolol: Increased QRS duration

All the associations listed are incorrect except verapamil (see Table 14-1). Because calcium blockers slow AV conduction, group 4 drugs such as verapamil and diltiazem increase PR interval and have little effect on the other ECG variables. The answer is C.

When working in outlying areas, this 62-year-old rancher is away from his house for 12-14 h at a time. He has an arrhythmia that requires chronic therapy. Which of the following has the longest half-life of all antiarrhythmic drugs? (A) Adenosine (B) Amiodarone (C) Disopyramide (D) Esmolol (E) Flecainide (F) Lidocaine (G) Mexiletine (H) Procainamide (I) Quinidine (J) Verapamil

Amiodarone has the longest half-life of all the antiarrhythmics (weeks). The answer is B.

Comparison of prazosin with atenolol shows that (A) Both decrease heart rate (B) Both increase cardiac output (C) Both increase renin secretion (D) Both increase sympathetic outflow from the CNS (E) Both produce orthostatic hypotension

Atenolol, but not prazosin, may decrease heart rate (choice A). Prazosin—but not atenolol—may increase cardiac output, a compensatory effect (choice B). Prazosin may increase renin output (a compensatory response), but β blockers inhibit its release by the kidney (choice C). By reducing blood pressure, both may increase central sympathetic outflow (a compensatory response). Beta blockers do not cause orthostatic hypotension. The answer is D.

A 36-year-old woman with a history of poorly controlled thyrotoxicosis has recurrent episodes of tachycardia with severe shortness of breath. When she is admitted to the emergency department with one of these episodes, which of the following drugs would be most suitable? (A) Amiodarone (B) Disopyramide (C) Esmolol (D) Quinidine (E) Verapamil

Beta blockers are the most effective agents in acute thyrotoxic arrhythmias. Esmolol is a parenteral, rapid-acting β blocker (see Chapter 10). The answer is C.

A 16-year-old girl has paroxysmal attacks of rapid heart rate with palpitations and shortness of breath. These episodes occasionally terminate spontaneously but often require a visit to the emergency department of the local hospital. Her ECG during these episodes reveals an AV nodal tachycardia. The antiarrhythmic of choice in most cases of acute AV nodal tachycardia is (A) Adenosine (B) Amiodarone (C) Flecainide (D) Propranolol (E) Verapamil

Calcium channel blockers are effective in supraventricular AV nodal tachycardias. However, adenosine is just as effective in most acute nodal tachycardias and is less toxic because of its extremely short duration of action. The answer is A.

A significant number of patients started on ACE inhibitor therapy for hypertension are intolerant and must be switched to a different class of drug. What is the most common manifestation of this intolerance? (A) Angioedema (B) Glaucoma (C) Headache (D) Incessant cough (E) Ventricular arrhythmias

Chronic, intolerable cough is an important adverse effect of captopril and other ACE inhibitors. It may be reduced or prevented by prior administration of aspirin. These drugs are very commonly used in hypertensive diabetic patients because of their proven benefits in reducing diabetic renal damage. The ACE inhibitors are not associated with glaucoma; angioedema is not as common as cough; and headache and arrhythmias are rare. The answer is D.

If this patient undergoes a percutaneous coronary angiography procedure and placement of a stent in a coronary blood vessel, he will need to be on dual antiplatelet therapy. eg, aspirin and clopidogrel for at least a year. Which of the following most accurately describes the mechanism of action of clopidogrel? (A) Clopidogrel directly binds to the platelet ADP receptors (B) Clopidogrel irreversibly inhibits cyclooxygenase (C) Clopidogrel facilitates the action of antithrombin III (D) The active metabolite of clopidogrel binds to the platelet ADP receptors (E) The active metabolite of clopidogrel binds to the platelet glycoprotein IIb/IIIa receptors

Clopidogrel is a prodrug that is activated by CYP2C9 and CYP2C19. It irreversibly binds to the ADP receptor on the surface of platelets that serves as a key role in platelet aggregation. Aspirin and clopidogrel help prevent platelet-induced occlusion of coronary stents. The answer is D.

Questions 1 and 2. A 76-year-old patient with rheumatoid arthritis and chronic heart disease is being considered for treatment with procainamide. She is already receiving digoxin, hydrochlorothiazide, and potassium supplements for her cardiac condition. 1. In deciding on a treatment regimen with procainamide for this patient, which of the following statements is most correct? (A) A possible drug interaction with digoxin suggests that digoxin blood levels should be obtained before and after starting procainamide (B) Hyperkalemia should be avoided to reduce the likelihood of procainamide toxicity (C) Procainamide cannot be used if the patient has asthma because it has a β-blocking effect (D) Procainamide cannot be used if the patient has angina because it has a β-agonist effect (E) Procainamide is not active by the oral route 2. If this patient should take an overdose and manifest severe acute procainamide toxicity with markedly prolonged QRS, which of the following should be given immediately? (A) A calcium chelator such as EDTA (B) Digitalis (C) Nitroprusside (D) Potassium chloride (E) Sodium lactate

1. Hyperkalemia facilitates procainamide toxicity. Procainamide is active by the oral route and has a duration of action of 2-4 h (in the prompt-release form). Procainamide has no significant documented interaction with digoxin and no significant β-agonist or β-blocking action. The answer is B. 2.The most effective therapy for procainamide toxicity appears to be concentrated sodium lactate. This drug may (1) increase sodium current by increasing the sodium ion gradient and (2) reduce drug-receptor binding by alkalinizing the tissue. The answer is E.

Questions 7-10. The matching questions in this section consist of a list of lettered options followed by several numbered items. For each numbered item, select the ONE lettered option that is most closely associated with it. Each lettered option may be selected once, more than once, or not at all. (A) Arsine (B) Deferoxamine (C) Dimercaprol (D) Edetate calcium disodium (E) Inorganic mercury (F) Iron (G) Methylmercury (H) Mercury vapor (I) Penicillamine (J) Succimer (K) Tetraethyl lead (L) Trivalent arsenic 7. This toxic compound can be produced in seawater by the action of bacteria and algae. It is also synthesized chemically for commercial use as a fungicide. 8. This agent is used in the treatment of Wilson's disease and has been reported to cause lupus erythematosus and hemolytic anemia. 9. High doses of this agent can cause histamine release and extreme vasodilation. 10. Gingivitis, discolored gums, and loose teeth are common symptoms of chronic exposure to this agent.

7. Methylmercury is used as a fungicide to prevent mold growth in seed grain. The answer is G. 8. Autoimmune diseases such as lupus erythematosus and hemolytic anemia have occurred during the treatment of Wilson's disease with penicillamine. The answer is I. 9. Deferoxamine can cause shock if given by rapid intravenous infusion. The answer is B. 10. Oral and gastrointestinal complaints are common in chronic mercury poisoning, and tremor involving the fingers and arms is often present. The answer is E.

Questions 8-10. The matching questions in this section consist of a list of lettered options followed by several numbered items. For each numbered item, select the ONE lettered option that is most closely associated with it. Each lettered option may be selected once, more than once, or not at all. (A) Aldicarb (B) Benzene (C) Carbon monoxide (D) Carbon dioxide (E) DDT (F) Dioxin (G) Malathion (H) Nitrogen dioxide (I) Paraquat (J) Pyrethrum (K) Rotenone (L) Sulfur dioxide (M) Tetrachloroethylene (N) Toluene 8. Asthma is often exacerbated in patients exposed to this reducing agent when concentrations in the air are as low as 1-2 ppm. It is formed mainly from combustion of fossil fuels. 9. Acute exposure to this aliphatic hydrocarbon solvent causes CNS depression; chronic exposure has led to impairment of memory and peripheral neuropathy. 10. This compound is a potential environmental hazard that is formed as a contaminating by-product in the manufacture of herbicides. It causes acneiform lesions and may be carcinogenic.

8. Sulfur dioxide is a reducing agent that forms sulfurous acid on contact with moist surfaces. This is responsible for irritant effects on mucous membranes of the eye, the oropharyngeal cavity, and the respiratory tract. Nitrogen dioxide causes similar problems, but it is an oxidizing agent formed from fires and in silage on farms. The answer is L. 9. Three hydrocarbon solvents are listed: benzene, tetrachloroethylene, and toluene. Each can cause CNS effects such as headache, fatigue, and loss of appetite. However, benzene and toluene are aromatic hydrocarbons. The answer is M. 10. Dioxin is a contaminant formed in the manufacture of chlorophenoxy acid herbicides, including 2,4-dichlorophenoxyacetic acid and 2,4,5-trichlorophenoxyacetic acid. The answer is F.

The patient is started on gemfibrozil. Which of the following is a major mechanism of gemfibrozil's action? (A) Increased excretion of bile acid salts (B) Increased expression of high-affinity LDL receptors (C) Increased secretion of VLDL by the liver (D) Increased triglyceride hydrolysis by lipoprotein lipase (E) Reduced uptake of dietary cholesterol

A major mechanism recognized for gemfibrozil is increased activity of the lipoprotein lipase associated with capillary endothelial cells. Gemfibrozil and other fibrates decrease VLDL secretion, presumably by stimulating hepatic fatty acid oxidation. The answer is D.

Which of the following is a major toxicity associated with gemfibrozil therapy? (A) Bloating and constipation (B) Cholelithiasis (C) Hyperuricemia (D) Liver damage (E) Severe cardiac arrhythmia

A major toxicity of the fibrates is increased risk of gallstone formation, which may be due to enhanced biliary excretion of cholesterol. The answer is B.

You are discussing the risks and benefits of anticoagulation therapy with her, including the option of using direct thrombin inhibitors. Which of the following anticoagulants is a direct inhibitor of thrombin? (A) Abciximab (B) Dabigatran (C) Rivaroxaban (D) Warfarin

Abciximab is an antiplatelet agent that binds to and inhibits GPIIb/IIIa. Rivaroxaban is an oral factor X inhibitor and warfarin inhibits vitamin K epoxide reductase (VKOR). The answer is B.

An 18-month-old boy presented in a semiconscious state with profound hypotension and bradycardia after ingesting a number of his grandmother's metoprolol tablets. In this case, the most appropriate antidote is (A) Atropine (B) Esmolol (C) Glucagon (D) Naloxone (E) Neostigmine

Glucagon (Chapter 41) stimulates heart rate and contractility through cardiac glucagon receptors that are coupled to adenylyl cyclase and the cAMP signaling pathway. This ability to increase cardiac cAMP without requiring access to β receptors makes it valuable for β-blocker overdose. The answer is C.

If this patient is pregnant, which of the following drugs should be avoided because of a risk of harming the fetus? (A) Cholestyramine (B) Ezetimibe (C) Fenofibrate (D) Niacin (E) Pravastatin

HMG-CoA reductase inhibitors are contraindicated in pregnancy because of the risk of teratogenic effects. The answer is E.

Which one of the following is a significant unwanted effect of the drug named? (A) Constipation with verapamil (B) Heart failure with hydralazine (C) Hemolytic anemia with atenolol (D) Hypokalemia with aliskiren (E) Lupus-like syndrome with hydrochlorothiazide

Hydralazine (choice B) is sometimes used in heart failure. Beta blockers (choice C) are not associated with hematologic abnormalities, but methyldopa is. The thiazide diuretics (choice E) often cause mild hyperglycemia, hyperuricemia, and hyperlipidemia but not lupus; hydralazine is associated with a lupus-like syndrome. Aliskiren (choice D) and other inhibitors of the renin-angiotensin-aldosterone system may cause hyperkalemia, not hypokalemia. Verapamil (choice A) often causes constipation, probably by blocking L-type calcium channels in the colon. The answer is A.

Another patient is admitted to the emergency department after a drug overdose. He is noted to have hypotension and severe bradycardia. He has been receiving therapy for hypertension and angina. Which of the following drugs in high doses causes bradycardia? (A) Amlodipine (B) Isosorbide dinitrate (C) Nitroglycerin (D) Prazosin (E) Verapamil

Isosorbide dinitrate (like all the nitrates) and prazosin can cause reflex tachycardia. Amlodipine, a dihydropyridine calcium channel blocker, causes much more vasodilation than cardiac depression and may also cause reflex tachycardia. Verapamil typically slows heart rate and high doses may cause severe bradycardia. The answer is E.

A 60-year-old man comes to the emergency department with severe chest pain. ECG reveals ventricular tachycardia with occasional normal sinus beats, and ST-segment changes suggestive of ischemia. A diagnosis of myocardial infarction is made, and the man is admitted to the cardiac intensive care unit. His arrhythmia should be treated immediately with (A) Adenosine (B) Digoxin (C) Lidocaine (D) Quinidine (E) Verapamil

Lidocaine has limited applications as an antiarrhythmic drug, but emergency treatment of myocardial infarction arrhythmias is one of the most important. Lidocaine is also useful in digoxin-induced arrhythmias. After recovery from the acute phase of a myocardial infarction, β blockers are used for 2 yr or more to prevent sudden death arrhythmias. The answer is C.

A 57-year-old man is admitted to the emergency department with chest pain and a fast irregular heart rhythm. The ECG shows an inferior myocardial infarction and ventricular tachycardia. Lidocaine is ordered. When used as an antiarrhythmic drug, lidocaine typically (A) Increases action potential duration (B) Increases contractility (C) Increases PR interval (D) Reduces abnormal automaticity (E) Reduces resting potential

Lidocaine reduces automaticity in the ventricles; the drug does not alter resting potential or AP duration and does not increase contractility. The answer is D.

Which one of the following drugs causes a decrease in liver triglyceride synthesis by limiting available free fatty acids needed as building blocks for this pathway? A. Niacin. B. Fenofibrate. C. Cholestyramine. D. Gemfibrozil. E. Lovastatin.

Correct answer = A. At gram doses, niacin strongly inhibits lipolysis in adipose tissue—the primary producer of circulating free fatty acids. The liver normally utilizes these circulating fatty acids as a major precursor for triglyceride synthesis. Thus, niacin causes a decrease in liver triglyceride synthesis, which is required for VLDL production. The other choices do not inhibit lipolysis in adipose tissue.

Which is most appropriate for reversing the anticoagulant effects of heparin? A. Aminocaproic acid. B. Protamine sulfate. C. Vitamin K1. D. Tranexamic acid.

Correct answer = B. Excessive bleeding may be managed by ceasing administration of heparin or by treating with protamine sulfate. Infused slowly, protamine sulfate combines ionically with heparin to form a stable, inactive complex. Aminocaproic acid and tranexamic acid are approved for the treatment of hemorrhage but do not specifically reverse the effects of heparin to stop bleeding. Vitamin K1 is used to help reverse the effects of warfarin-induced bleeding.

Which is important to monitor in patients taking digoxin? A. Chloride. B. Potassium. C. Sodium. D. Zinc.

Correct answer = B. Hypokalemia can lead to life-threatening arrhythmias and increases the potential of cardiac toxicity with digoxin.

A 3-year-old boy is brought to the emergency department by his mother, who reports that he has been crying continuously and "does not want to play or eat" for the last few days. She also states that he has not had regular bowel movements, with mostly constipation and occasional diarrhea, and frequently complains of abdominal pain. The child now has an altered level of consciousness, is difficult to arouse, and begins to seize. The clinician rules out infection and other medical causes. Upon questioning, the mother states that the house is in an older neighborhood, that her house has not been remodeled or repainted since the 1940s, and that the paint is chipping around the windows and doors. The child is otherwise breathing on his own and urinating normally. Which toxin would you expect to be producing such severe effects in this child? A. Iron. B. Lead. C. Carbon monoxide. D. Cyanide. E. Ethylene glycol.

Correct answer = B. Lead poisoning is common among children in older homes painted before lead was removed from paint. Paint chips with lead are easily ingested by toddlers, and excessively high lead levels can lead to the signs and symptoms described plus clumsiness, confusion, headaches, coma, constipation, intestinal spasms, and anemia. Death is rare when chelation therapy is instituted. Iron can produce abdominal pain, but more often would cause diarrhea, vomiting, and volume loss. If he had cyanide poisoning, death would have occurred quickly following respiratory arrest of oxidative phosphorylation and production of adenosine triphosphate, but this child has been exhibiting symptoms over several days. Carbon monoxide would affect the entire household, depending on the source. Clinical effects from carbon monoxide would include headache, nausea, and CNS depression. Ethylene glycol is sweet and may be ingested by a toddler. The presentation of ethylene glycol toxicity would include initial appearance of intoxication, which was not mentioned.

What makes losartan different from other ARBs? A. Losartan is renally eliminated. B. Losartan has an active metabolite. C. Losartan has the shortest half-life. D. Losartan has a small volume of distribution

Correct answer = B. Losartan is the only ARB that undergoes first-pass metabolism to convert to its active metabolite. Most ARBs have once-daily dosing, and all (except candesartan) have large volumes of distribution.

Which describes the mechanism of action of milrinone in HF? A. Decreases intracellular calcium. B. Increases cardiac contractility. C. Decreases cAMP. D. Activates phosphodiesterase.

Correct answer = B. Milrinone is a phosphodiesterase inhibitor that leads to increased cAMP, increased intracellular calcium, and therefore increased contractility.

Which statement regarding dronedarone is correct? A. Dronedarone is more effective than amiodarone. B. QT interval prolongation is not a risk with dronedarone. C. Dronedarone increases the risk of death in patients with permanent atrial fibrillation or symptomatic heart failure. D. There is no need to monitor liver function with dronedarone.

Correct answer = C. Dronedarone is not as effective as amiodarone, QT prolongation is a risk with this drug, and liver function should be monitored when taking dronedarone since it increases the risk of liver failure. The drug is contraindicated in those with symptomatic heart failure or permanent atrial fibrillation due to an increased risk of death.

Which side effect is associated with amlodipine? A. Bradycardia. B. Cough. C. Edema. D. QT prolongation.

Correct answer = C. Edema is the correct answer. The other answers are incorrect.

A 40-year-old male has recently been diagnosed with hypertension due to pressure readings of 163/102 and 165/100 mm Hg. He also has diabetes that is well controlled with oral hypoglycemic medications. Which is the best initial treatment regimen for treatment of hypertension in this patient? A. Felodipine. B. Furosemide. C. Lisinopril. D. Lisinopril and hydrochlorothiazide. E. Metoprolol.

Correct answer = D. Because the systolic blood pressure is more than 20 mm Hg above goal (10 mm Hg above goal diastolic), treatment with two different medications is preferred. Because the patient is diabetic, he also has a compelling indication for an ACE inhibitor or ARB.

A 72-year-old male presents to the primary care clinic complaining of chest tightness and pressure that is increasing in severity and frequency. His current medications include atenolol, lisinopril, and nitroglycerin. Which intervention is most appropriate at this time? A. Add amlodipine. B. Initiate isosorbide mononitrate. C. Initiate ranolazine. D. Refer the patient to the nearest emergency room for evaluation.

Correct answer = D. Crescendo angina is indicative of unstable angina that requires further workup.

Which of the following is correct regarding digoxin when used for atrial fibrillation? A. Digoxin works by blocking voltage-sensitive calcium channels. B. Digoxin is used for rhythm control in patients with atrial fibrillation. C. Digoxin increases conduction velocity through the AV node. D. Digoxin levels of 1 to 2 ng/mL are desirable in the treatment of atrial fibrillation.

Correct answer = D. Digoxin works by inhibiting the Na+/K+ATPase pump. It decreases conduction velocity through the AV node and is used for rate control in atrial fibrillation (not rhythm control). Digoxin levels between 1 and 2 ng/mL are more likely to exhibit negative chronotropic effects desired in atrial fibrillation or flutter. A serum drug concentration between 0.5 and 0.8 ng/mL is for symptomatic management of heart failure.

A 60-year-old white female has not reached her blood pressure goal after 1 month of treatment with a low dose of lisinopril. All of the following would be appropriate next steps in the treatment of her hypertension except: A. Increase dose of lisinopril. B. Add a diuretic medication. C. Add on a calcium channel blocker medication. D. Add on an ARB medication

Correct answer = D. Increasing the dose of lisinopril or adding a second medication from a different class (such as a calcium channel blocker or diuretic) would be appropriate steps to control the blood pressure. Adding an ARB as the second medication is not recommended. ARBs have a similar mechanism of action to ACE inhibitors, and combination therapy may increase the risk of adverse effects.

Which arrhythmia can be treated with lidocaine? A. Paroxysmal supraventricular ventricular tachycardia. B. Atrial fibrillation. C. Atrial flutter. D. Ventricular tachycardia.

Correct answer = D. Lidocaine has little effect on atrial or AV nodal tissue; thus, it used for ventricular arrhythmias such as ventricular tachycardia.

A patient returns to her health care provider for routine monitoring 3 months after her hypertension regimen was modified. Labs reveal elevated serum potassium. Which is likely responsible for this hyperkalemia? A. Chlorthalidone. B. Clonidine. C. Furosemide. D. Losartan. E. Nifedipine.

Correct answer = D. Losartan, an ARB, can cause an increase in serum potassium similar to ACE inhibitors. Furosemide and chlorthalidone can cause a decrease in serum potassium. Nifedipine and clonidine do not affect potassium levels.

Which one of the following drugs decreases cholesterol synthesis by inhibiting the enzyme 3-hydroxy-3methylglutaryl coenzyme A reductase? A. Fenofibrate. B. Niacin. C. Cholestyramine. D. Lovastatin. E. Gemfibrozil.

Correct answer = D. Lovastatin decreases cholesterol synthesis by inhibiting HMG CoA reductase. Fenofibrate and gemfibrozil increase the activity of lipoprotein lipase, thereby increasing the removal of VLDL from plasma. Niacin inhibits lipolysis in adipose tissue, thus eliminating the building blocks needed by the liver to produce triglycerides and, therefore, VLDL. Cholestyramine lowers the amount of bile acids returning to the liver via the enterohepatic circulation.

BC is a 70-year-old female who is diagnosed with HFrEF. Her past medical history is significant for hypertension and atrial fibrillation. She is taking hydrochlorothiazide, lisinopril, metoprolol tartrate, and warfarin. BC says she is feeling "good" and has no cough, shortness of breath, or edema. Which is the most appropriate medication change to make? A. Discontinue hydrochlorothiazide. B. Change lisinopril to losartan. C. Decrease warfarin dose. D. Change metoprolol tartrate to metoprolol succinate

Correct answer = D. Metoprolol succinate should be used in HF, given that there is mortality benefit shown with metoprolol succinate in landmark HF trials. Hydrochlorothiazide and warfarin are appropriate based on the information given; there is no reason to change to an ARB since the patient has no cough or history of angioedema.

Which drug may exacerbate HF? A. Acetaminophen. B. Cetirizine. C. Chlorthalidone. D. Ibuprofen.

Correct answer = D. NSAIDs, such as ibuprofen, lead to increased fluid retention and increased blood pressure. If possible, NSAIDs should be avoided in HF patients in order to avoid exacerbations of HF

A 4-year-old female presents to the emergency department with CNS depression. Her vital signs indicate that she is slightly bradycardic and slightly hypotensive for her age. Upon further questioning, the mother admits that there are two clonidine 0.2 mg tablets missing from the home. Which of the following antidotes might be beneficial for this patient? A. Flumazenil. B. Atropine. C. Deferoxamine. D. Naloxone. E. Succimer.

Correct answer = D. Naloxone has a reversal rate of the CNS effects of approximately 50% in clonidine ingestions. Flumazenil reverses benzodiazepines and has no effect on clonidine. Atropine is an anticholinergic agent and would not improve the CNS depression. Deferoxamine is the chelator for iron, and succimer is a lead chelator

A 68-year-old male with a history of angina had a MI last month, and an echocardiogram reveals heart failure with reduced ejection fraction. He was continued on his previous home medications (diltiazem, enalapril, and nitroglycerin), and atenolol was added at discharge. He has only had a few sporadic episodes of stable angina that are relieved with nitroglycerin or rest. What are eventual goals for optimizing this medication regimen? A. Add isosorbide mononitrate. B. Increase atenolol. C. Stop atenolol and increase diltiazem. D. Stop diltiazem and change atenolol to bisoprolol.

Correct answer = D. Nondihydropyridine calcium channel blockers such as diltiazem should be avoided in patients with heart failure with reduced ejection fraction. Patients should be treated with one of three β-blockers approved for heart failure with reduced ejection fraction (bisoprolol, metoprolol succinate, or carvedilol). It sounds like his angina symptoms are well managed with his current therapy so adding isosorbide mononitrate would not be necessary. These symptoms may become even less frequent as his new β-blocker is titrated.

Which may cause reflex tachycardia and/or postural hypotension on initial administration? A. Atenolol. B. Hydrochlorothiazide. C. Metoprolol. D. Prazosin. E. Verapamil.

Correct answer = D. Prazosin produces first-dose hypotension, presumably by blocking α1 receptors. This effect is minimized by initially giving the drug in small, divided doses. The other agents do not have this adverse effect.

JS is a 65-year-old man who presents to his physician for management of hyperlipidemia. His most recent lipid panel reveals an LDL cholesterol level of 165 mg/ dL. His physician wishes to begin treatment to lower his LDL cholesterol levels. Which of the following therapies is the best option to lower JS's LDL cholesterol levels? A. Fenofibrate. B. Colesevelam. C. Niacin. D. Simvastatin. E. Ezetimibe.

Correct answer = D. Simvastatin, an HMG CoA reductase inhibitor (statin), is the most effective option for lowering LDL cholesterol, achieving reductions of 30% to 41% from baseline levels. Fenofibrate and niacin are more effective at lowering triglyceride levels or raising HDL levels (niacin). Colesevelam can reduce LDL levels but not as effectively as statins. Ezetimibe lowers LDL levels modestly compared to the LDL reduction achieved by statins.

A 5-year-old male is brought in to the health care facility for being irritable and failure to thrive. He is alert, and his vital signs are normal. The doctor diagnoses him with lead toxicity when the blood lead level returns as 50 μg/dL. Which chelator regimen should be started? A. Dimercaprol. B. Calcium disodium edetate. C. Both dimercaprol and calcium disodium edetate. D. Succimer. E. Deferoxamine.

Correct answer = D. Succimer (dimercaptosuccinic acid, DMSA) is utilized when the lead level is greater than 45 μg/dL, without encephalopathy. If encephalopathy is present, or the lead level is greater than 70 μg/dL in a child, then dual parenteral therapy with dimercaprol and calcium disodium edetate is indicated. Dimercaprol intramuscular therapy is initiated 4 hours prior to the intravenous administration of calcium disodium edetate when both medications are required. Deferoxamine is not indicated since it is the chelator for iron.

All of the following are reasonable combinations of immunosuppressive drugs except: A. Basiliximab, belatacept, mycophenolate mofetil, and prednisone. B. Thymoglobulin, cyclosporine, azathioprine, and prednisone. C. Tacrolimus, mycophenolate mofetil, and prednisone. D. Tacrolimus, cyclosporine, and prednisone. E. Tacrolimus, sirolimus, and prednisone.

Correct answer = D. Tacrolimus and cyclosporine are both calcineurin inhibitors and have the same mechanism of action. Immunosuppressive drug regimens should work synergistically at different places in the T-cell activation cascade. Additionally, cyclosporine and tacrolimus are both extremely nephrotoxic and when used together would cause harm to the patients.

A 70-year-old female is diagnosed with nonvalvular atrial fibrillation. Her past medical history is significant for chronic kidney disease, and her renal function is moderately diminished. All of the following anticoagulants would be expected to require a reduced dosage in this patient except: A. Apixaban. B. Dabigatran. C. Rivaroxaban. D. Warfarin.

Correct answer = D. Warfarin does not require dosage adjustment in renal dysfunction. The INR is monitored and dosage adjustments are made on the basis of this information. All of the other agents are renally cleared to some extent and require dosage adjustments in renal dysfunction

A 78-year-old woman has been newly diagnosed with atrial fibrillation. She is not currently having symptoms of palpitations or fatigue. Which is appropriate to initiate for rate control as an outpatient? A. Amiodarone. B. Dronedarone. C. Esmolol. D. Flecainide. E. Metoprolol.

Correct answer = E. Only C and E are options to control rate. The other options are used for rhythm control in patients with atrial fibrillation. Since esmolol is IV only, the only option to start as an outpatient is metoprolol.

Which of the following drugs used to prevent allograft rejection can cause hyperlipidemia? A. Azathioprine. B. Basiliximab. C. Belatacept. D. Mycophenolate mofetil. E. Sirolimus.

Correct answer = E. Patients who are receiving sirolimus can develop elevated cholesterol and triglyceride levels, which can be controlled by statin therapy. None of the other agents has this adverse effect.

Although sirolimus and cyclosporine have similar immunosuppressant effects, their toxicity profiles differ. Which of the following toxicities is more likely to be associated with sirolimus than with cyclosporine? (A) An anaphylactic reaction (B) Hypertension (C) Osteoporosis (D) Renal insufficiency (E) Thrombocytopenia

Cyclosporine and tacrolimus both are associated with renal toxicity and hypertension. In contrast, sirolimus appears to spare the kidney and instead is more likely to cause gastrointestinal disturbance, hypertriglyceridemia, and myelosuppression, especially in the form of thrombocytopenia. The answer is E.

Which of the following drugs slows conduction through the AV node and has its primary action directly on L-type calcium channels? (A) Adenosine (B) Amiodarone (C) Diltiazem (D) Esmolol (E) Flecainide (F) Lidocaine (G) Mexiletine (H) Procainamide (I) Quinidine

Diltiazem is the calcium channel blocker in this list. (Beta blockers also slow AV conduction but have much smaller effects on calcium channels.) The answer is C.

A 73-year-old man with a history of a recent change in his treatment for moderately severe hypertension is brought to the emergency department because of a fall at home. Which of the following drug groups is most likely to cause postural hypotension and thus an increased risk of falls? (A) ACE inhibitors (B) Alpha1-selective receptor blockers (C) Arteriolar dilators (D) Beta1-selective receptor blockers (E) Nonselective β blockers

Drug-induced postural (orthostatic) hypotension is usually due to venous pooling or excessive diuresis and inadequate blood volume. Venous pooling is normally prevented by α-receptor activation in vascular smooth muscle; thus, orthostatic hypotension is caused or exacerbated by α1 blockers, eg, prazosin. The answer is B.

She tells you that her main reason for not wanting oral anticoagulation is that she does not want to come to clinic for frequent blood draws. You agree on an oral alternative and start her on apixaban. You counsel her extensively on the importance of taking the medication each day, as suddenly stopping can lead to (A) Anaphylaxis (B) Excess bleeding (C) Increase in INR (D) Stroke (E) Thrombocytopenia

Due to the shorter half-life of the oral factor X and thrombin inhibitors, the anticoagulant status of the patient changes rapidly. Sudden cessation of short-acting oral anticoagulants can lead to stroke. Excess bleeding is associated with taking any of the anticoagulants not with stopping them. An increase in INR reflects increased anticoagulation by warfarin. Thrombocytopenia is a risk associated with heparin. The answer is D.

The decision was made to treat this woman with enoxaparin. Relative to unfractionated heparin, enoxaparin (A) Can be used without monitoring the patient's aPTT (B) Has a shorter duration of action (C) Is less likely to have a teratogenic effect (D) Is more likely to be given intravenously (E) Is more likely to cause thrombosis and thrombocytopenia

Enoxaparin is an LMW heparin. LMW heparins have a longer half-life than standard heparin and a more consistent relationship between dose and therapeutic effect. Enoxaparin is given subcutaneously, not intravenously. It is less, not more, likely to cause thrombosis and thrombocytopenia. Neither LMW heparins nor standard heparin are teratogenic. The aPTT is not useful for monitoring the effects of LMW heparins. The answer is A.

A patient with heart failure has accidentally taken an overdose of digoxin. The blood concentration of the drug is 8 times the threshold for toxicity. Pharmacokinetic parameters for digoxin in this patient include a clearance of 7 L/h and an elimination half-life of 56 h. If no procedures are instituted to decontaminate this patient, the time taken to reach a safe level of digoxin will be approximately (A) 3.5 d (B) 7 d (C) 14 d (D) 28 d (E) 56 d

Estimations of the time period required for drug or toxin elimination may be of value in the management of the poisoned patient. If no procedures were used to hasten the elimination of digoxin in this patient, the time taken to reach a safe plasma level of the drug (12.5% of the measured level) is 3 half-lives, or approximately 7 d. The answer is B.

A 45-year-old man is brought to the emergency department with mental obtundation. He is found to have a blood pressure of 220/160 and retinal hemorrhages. Which one of the following is used in severe hypertensive emergencies, is shortacting, acts on a G protein-coupled receptor, and must be given by intravenous infusion? (A) Aliskiren (B) Captopril (C) Fenoldopam (D) Hydralazine (E) Losartan (F) Metoprolol (G) Nitroprusside (H) Prazosin (I) Propranolol

Fenoldopam, nitroprusside, and propranolol are the drugs in the list that have been used in hypertensive emergencies. Fenoldopam and nitroprusside are used by infusion only, but nitroprusside releases nitric oxide, which acts on intracellular guanylyl cyclase. The answer is C.

A 24-year-old woman underwent kidney transplantation. A week later, she developed alloantibody-mediated acute rejection (acute humoral rejection [AHR]). She was successfully treated with tacrolimus and a second drug that targets both B and T lymphocytes. Which of the following is an immunosuppressant that suppresses both B and T lymphocytes via inhibition of de novo synthesis of purines? (A) Cyclophosphamide (B) Methotrexate (C) Mycophenolate mofetil (D) Prednisone (E) Tacrolimus

Mycophenolic acid, formed from mycophenolate mofetil, inhibits inosine monophosphate dehydrogenase, the rate-limiting enzyme in the de novo pathway of purine synthesis. This action suppresses both B- and T-lymphocyte activation. Mycophenolate mofetil is used in organ transplantation. The answer is C.

A contraindication to the use of gastric lavage for the removal of drugs from the stomach of victim of poisoning is (A) An overdose of iron pills (B) An unconscious patient (C) Ingestion of a corrosive (D) Overdose with a sustained-release formulation

Neither gastric lavage nor syrup of ipecac should be used in patients who have ingested a corrosive because of the risk of esophageal damage. Gastric lavage can be used in a comatose patient if the airway has been protected with a cuffed endotracheal tube. The answer is C.

A 72-year-old woman has long-standing heart failure. Which one of the following drugs has been shown to reduce mortality in chronic heart failure? (A) Atenolol (B) Digoxin (C) Dobutamine (D) Furosemide (E) Spironolactone

Of the drugs listed, only spironolactone has been shown to reduce mortality in this highly lethal disease. Digoxin, dobutamine, and furosemide are used in the management of symptoms. The answer is E.

A farm worker was accidentally in the field during the aerial spraying with parathion. He was brought to the emergency department. Which of the following will be used in the treatment of this patient? (A) Antiseizure drugs (B) Atropine and pralidoxime (C) Hemodialysis (D) Hyperbaric oxygen (E) Measures to reduce pulmonary edema

Organophosphate poisoning is treated with the muscarinic receptor antagonist atropine and pralidoxime, which regenerates cholinesterase. The answer is B.

. If a β blocker were to be used for prophylaxis in this patient, what is the most probable mechanism of action in angina? (A) Block of exercise-induced tachycardia (B) Decreased end-diastolic ventricular volume (C) Increased double product (D) Increased cardiac force (E) Decreased ventricular ejection time

Propranolol blocks tachycardia but has none of the other effects listed. Only revascularization increases double product; drugs that decrease cardiac work increase exercise time by decreasing double product. The answer is A.

A man is admitted to the emergency department with a brownish cyanotic appearance, marked shortness of breath, and hypotension. Which of the following is most likely to cause methemoglobinemia? (A) Amyl nitrite (B) Isosorbide dinitrate (C) Isosorbide mononitrate (D) Nitroglycerin (E) Sodium cyanide

Read carefully! Nitrites, not nitrates, cause methemoglobinemia in adults. Methemoglobinemia is delibrately induced in one of the treatments of cyanide poisoning. The answer is A.

Which of the following drugs accelerates the conversion of plasminogen to plasmin? (A) Aminocaproic acid (B) Heparin (C) Argatroban (D) Reteplase (E) Warfarin

Reteplase is the only thrombolytic drug listed. Heparin and warfarin are anticoagulants. Argatroban is a direct inhibitor of thrombin, and aminocaproic acid is an inhibitor, not an activator, of the conversion of plasminogen to plasmin. The answer is D.

A 30-year-old woman has one living child, age 6 years. Her child and her husband are Rh positive and she is Rho(D) and Du negative. She is now in her ninth month of pregnancy and is in the labor room having frequent contractions. Her Rh antibody test taken earlier in the pregnancy was negative. What immunotherapy is appropriate for this patient? (A) Cyclosporine (B) Cyclophosphamide (C) Methotrexate (D) Rho(D) immune globulin (E) Tacrolimus

Rho(D) immune globulin contains antibodies against Rho(D) antigens. If an injection of Rho(D) antibody is administered to the Rh-negative mother within 24-72 h after the birth of an Rh-positive infant, the mother's own antibody response to the foreign Rho(D)-positive cells is suppressed because the infant's red cells are cleared from circulation before the mother can generate a B-cell response against Rho(D). Therefore, she has no memory B cells that can activate upon subsequent pregnancies with an Rho(D)positive fetus. The answer is D.

Which of the following has been shown to prolong life in patients with chronic congestive failure in spite of having a negative inotropic effect on cardiac contractility? (A) Carvedilol (B) Digoxin (C) Dobutamine (D) Enalapril (E) Furosemide

Several β blockers, including carvedilol, have been shown to prolong life in heart failure patients even though these drugs have a negative inotropic action on the heart. Their benefits presumably result from some other effect, and at least one β blocker has failed to show a mortality benefit. The answer is A.

In advising the patient about the adverse effects he may notice, you point out that nitroglycerin in moderate doses often produces certain symptoms. Which of the following effects might occur due to the mechanism listed? (A) Constipation (B) Dizziness due to reduced cardiac force of contraction (C) Diuresis due to sympathetic discharge (D) Headache due to meningeal vasodilation (E) Hypertension due to reflex tachycardia

The nitrates relax many types of smooth muscle, but the effect on motility in the colon is insignificant. Nitroglycerin causes hypotension as a result of arterial and venous dilation. Dilation of arteries in the meninges has no effect on central nervous system function but does cause headache. The answer is D.

If a fibrinolytic drug is used for treatment of this man's acute myocardial infarction, which of the following adverse drug effects is most likely to occur? (A) Acute renal failure (B) Development of antiplatelet antibodies (C) Encephalitis secondary to liver dysfunction (D) Hemorrhagic stroke (E) Neutropenia

The most common serious adverse effect of the fibrinolytics is bleeding, especially in the cerebral circulation. The fibrinolytics do not usually have serious effects on the renal, hepatic, or hematologic systems. Unlike heparin, they do not induce antiplatelet antibodies. The answer is D.

A 24-year-old female was rushed to the emergency department after she was found in her room hypotensive, with seizures. In the emergency department, the electrocardiogram confirmed ventricular arrhythmias. An overdose of which of the following drugs is the most likely cause of her symptoms? (A) Acetaminophen (B) Amitriptyline (C) Diazepam (D) Ethylene glycol (E) Morphine

Tricyclic antidepressants such as amitriptyline are extremely toxic in overdose because of their effects in the CNS and cardiovascular system. In addition to hypotension, seizures, and cardiac arrhythmias, the tricyclics have strong antimuscarinic effects. The answer is B.

A patient is admitted to the emergency department with severe tachycardia after a drug overdose. His family reports that he has been depressed about his hypertension. Which one of the following drugs increases the heart rate in a dosedependent manner? (A) Captopril (B) Hydrochlorothiazide (C) Losartan (D) Minoxidil (E) Verapamil

ACE inhibitors (choice A), ARBs (choice C), and diuretics (choice B) do not significantly increase heart rate. Although dihydropyridine calcium channel blockers do not usually reduce rate markedly (and may increase it), verapamil (choice E) and diltiazem do inhibit the sinoatrial node and predictably decrease rate. Other direct vasodilators (choice D) regularly increase heart rate, and minoxidil, a very efficacious vasodilator, causes severe tachycardia that must be controlled with β blockers. The answer is D.

A 65-year-old woman has been admitted to the coronary care unit with a left ventricular myocardial infarction. She develops acute severe heart failure with marked pulmonary edema, but no evidence of peripheral edema. Which one of the following drugs would be most useful? (A) Digoxin (B) Furosemide (C) Minoxidil (D) Propranolol (E) Spironolactone

Acute severe congestive failure with pulmonary edema often requires a vasodilator that reduces intravascular pressures in the lungs. Furosemide has such vasodilating actions in the context of acute failure. Pulmonary edema also involves a shift of fluid from the intravascular compartment to the lungs. Minoxidil would decrease arterial pressure and increase the heart rate excessively. Digoxin has a slow onset of action and lacks vasodilating effects. Spironolactone is useful in chronic failure but not in acute pulmonary edema. Pulmonary vasodilation and removal of edema fluid by diuresis are accomplished by furosemide. The answer is B.

A new 60-year-old patient presents to the medical clinic with hypertension and angina. He is 1.8 meters tall with a waist measurement of 1.1 m. Weight is 97 kg. Blood pressure is 150/95 and pulse 85. In considering adverse effects of possible drugs for these conditions, you note that an adverse effect that nitroglycerin and prazosin have in common is (A) Bradycardia (B) Impaired sexual function (C) Lupus erythematosus syndrome (D) Orthostatic hypotension (E) Weight gain

Both drugs cause venodilation and reduce venous return sufficiently to cause some degree of postural hypotension. Bradycardia, lupus, weight gain, and urinary retention occur with neither of them, but prazosin has been used to relieve urinary retention in men with prostatic hyperplasia. The answer is D.

How do β-blockers improve cardiac function in HF? A. By decreasing cardiac remodeling. B. By increasing heart rate. C. By increasing renin release. D. By activating norepinephrine.

Correct answer = A. Although it seems counterintuitive to decrease heart rate in HF, β-blockers improve cardiac functioning by slowing heart rate, decreasing renin release, and preventing the direct effects of norepinephrine on cardiac muscle to decrease remodeling.

A 58-year-old man receives intravenous alteplase treatment for acute stroke. Five minutes following completion of alteplase infusion, he develops orolingual angioedema. Which of the following drugs may have increased the risk of developing orolingual angioedema in this patient? A. ACE inhibitor. B. GP IIb/IIIa receptor antagonist. C. Phosphodiesterase inhibitor. D. Thiazide diuretic.

Correct answer = A. ACE inhibitors, aspirin, and prasugrel all have possible adverse effects including orolingual angioedema. In the setting of alteplase administration, ACE inhibitors have been associated with an increased risk of developing orolingual angioedema with concomitant use.

Which is considered "fibrin selective" because it rapidly activates plasminogen that is bound to fibrin? A. Alteplase. B. Fondaparinux. C. Streptokinase. D. Urokinase.

Correct answer = A. Alteplase has a low affinity for free plasminogen in the plasma, but it rapidly activates plasminogen that is bound to fibrin in a thrombus or a hemostatic plug. It has the advantage of lysing only fibrin, without unwanted degradation of other proteins (notably fibrinogen).

DD is a 50-year-old male with newly diagnosed hypertension. His comorbidities include diabetes and chronic hepatitis C infection with moderate liver impairment. He requires two drugs for initial treatment of his hypertension. Which should be prescribed in combination with a thiazide diuretic? A. Lisinopril. B. Spironolactone. C. Fosinopril. D. Furosemide. E. Hydralazine.

Correct answer = A. Because DD has diabetes, he has a compelling indication for an ACE inhibitor or ARB for the treatment of his hypertension and prevention of diabetic nephropathy. However, most ACE inhibitors undergo hepatic conversion to active metabolites, so his hepatic impairment is of concern. Because lisinopril is one of the two ACE inhibitors that does not undergo hepatic conversion to active metabolites, it is the best choice. Fosinopril is the only ACE inhibitor that is not eliminated primarily by the kidneys but does undergo hepatic conversion. An additional diuretic like spironolactone or furosemide is not indicated. DD does not have a compelling indication for hydralazine.

AJ is a 42-year-old man who was started on niacin sustained-release tablets 2 weeks ago for elevated triglycerides and low HDL levels. He is complaining of an uncomfortable flushing and itchy feeling that he thinks is related to the niacin. Which of the following options can help AJ manage this adverse effect of niacin therapy? A. Administer aspirin 30 minutes prior to taking niacin. B. Administer aspirin 30 minutes after taking niacin. C. Increase the dose of niacin SR to 1000 mg. D. Continue the current dose of niacin. E. Change the sustained-release niacin to immediate-release niacin.

Correct answer = A. Flushing associated with niacin is prostaglandin mediated; therefore, use of aspirin (a prostaglandin inhibitor) can help to minimize this adverse effect. It must be administered 30 minutes prior to the dose of the niacin; therefore, choice B is incorrect. Increasing the dose of niacin is likely to increase these complaints; therefore, choice C is incorrect. Continuing the current dose is unlikely to relieve these complaints, which are bothersome to AJ. The sustained-release formulation of niacin has less incidence of flushing versus that of the immediate release; therefore, choice E is incorrect.

3 Which can precipitate a hypertensive crisis following abrupt cessation of therapy? A. Clonidine. B. Diltiazem. C. Enalapril. D. Losartan. E. Hydrochlorothiazide.

Correct answer = A. Increased sympathetic nervous system activity occurs if clonidine therapy is abruptly stopped after prolonged administration. Uncontrolled elevation in blood pressure can occur. Patients should be slowly weaned from clonidine while other antihypertensive medications are initiated. The other drugs on the list do not produce this phenomenon.

A clinician would like to initiate a drug for rhythm control of atrial fibrillation. Which of the following coexisting conditions would allow for initiation of flecainide? A. Hypertension. B. Left ventricular hypertrophy. C. Coronary artery disease. D. Heart failure.

Correct answer = A. Since flecainide can increase the risk of sudden cardiac death in those with a history of structural heart disease, only A will allow for flecainide initiation. Structural heart disease includes left ventricular hypertrophy, heart failure, and atherosclerotic heart disease.

A 45-year-old man was just started on therapy for hypertension and developed a persistent, dry cough. Which is most likely responsible for this side effect? A. Enalapril. B. Losartan. C. Nifedipine. D. Prazosin. E. Propranolol

Correct answer = A. The cough is most likely an adverse effect of the ACE inhibitor enalapril. Losartan is an ARB that has the same beneficial effects as an ACE inhibitor but is less likely to produce a cough. Nifedipine, prazosin, and propranolol do not cause this side effect.

Which one of the following hyperlipidemias is characterized by elevated plasma levels of chylomicrons and has no drug therapy available to lower the plasma lipoprotein levels? A. Type I. B. Type II. C. Type III. D. Type IV. E. Type V.

Correct answer = A. Type I hyperlipidemia (hyperchylomicronemia) is treated with a low-fat diet. No drug therapy is effective for this disorder.

A 62-year-old patient with a history of asthma and vasospastic angina states that he gets chest pain both with exertion and at rest, about ten times per week. One sublingual nitroglycerin tablet always relieves his symptoms, but this medication gives him an awful headache every time he takes it. Which is the best option for improving his angina? A. Change to sublingual nitroglycerin spray. B. Add amlodipine. C. Add propranolol. D. Replace nitroglycerin with ranolazine.

Correct answer = B. Calcium channel blockers are preferred for vasospastic angina. β-Blockers can actually worsen vasospastic angina; furthermore, nonselective β-blockers should be avoided in patients with asthma. The nitroglycerin spray would also be expected to cause headache, so this is not the best choice. Ranolazine is not indicated for immediate relief of an angina attack, nor is it a first-line option.

WW is a 62-year-old female with hyperlipidemia and hypothyroidism. Her current medications include cholestyramine and levothyroxine (thyroid hormone). What advice would you give to WW to avoid a drug interaction between her cholestyramine and levothyroxine? A. Stop taking the levothyroxine as it can interact with cholestyramine. B. Take levothyroxine 1 hour before cholestyramine on an empty stomach. C. Switch cholestyramine to colesevelam as this will eliminate the interaction. D. Switch cholestyramine to colestipol as this will eliminate the interaction. E. Take levothyroxine and cholestyramine at the same time to minimize the interaction.

Correct answer = B. Cholestyramine and the bile acid resins can bind several medications causing decreased absorption. Cholestyramine can decrease absorption of medications such as levothyroxine. Taking levothyroxine 1 hour before or 4 to 6 hours after cholestyramine can help to avoid this interaction. Choices C and D are incorrect, as all bile acid resins cause this interaction. Choice A is incorrect, as this patient should not stop her thyroid medication. Choice E will worsen this drug interaction.

A 56-year-old man presents to the emergency room with complaints of swelling, redness, and pain in his right leg. The patient is diagnosed with acute DVT and requires treatment with an anticoagulant. All of the following are approved for treatment of this patient's DVT except: A. Rivaroxaban. B. Dabigatran. C. Enoxaparin. D. Heparin.

Correct answer = B. Dabigatran is only approved for the prevention of stroke in nonvalvular atrial fibrillation; it is not approved for the treatment of acute DVT. All of the other options are approved for treatment of acute DVT.

A 27-year-old female presents to the emergency department 6 hours after reportedly ingesting 20 tablets of acetaminophen 500 mg. An acetaminophen level is drawn, but it has to be sent out to another lab and will not return for another 6 hours. What is the most appropriate next step in management of this patient? A. Administer a dose (50 g) of activated charcoal. B. Empirically start N-acetylcysteine therapy. C. Administer a dose of intravenous naloxone. D. Wait for the level to return and then decide what to do. E. Draw a NAPQI level.

Correct answer = B. N-acetylcysteine should be started empirically on the basis of the history, and then, once the level returns and is plotted on the Rumack-Matthew nomogram, a final decision on whether to continue therapy can be made. Activated charcoal would not be of any benefit 6 hours post-acetaminophen ingestion. Naloxone is utilized for opioid toxicity, not acetaminophen toxicity. The optimal time frame to give N-acetylcysteine is within 8 to 10 hours postingestion. So, waiting on the level to return would put the patient more than 12 hours postingestion. Therefore, initiation of N-acetylcysteine therapy should happen, if possible during the optimal time frame. Clinicians are unable to draw a NAPQI level and therefore cannot utilize this to guide therapy.

Which of the following patient populations is more likely to experience myalgia (muscle pain) or myopathy with use of HMG CoA reductase inhibitors? A. Patients with diabetes mellitus. B. Patients with renal insufficiency. C. Patients with gout. D. Patients with hypertriglyceridemia. E. Patients taking warfarin (blood thinner).

Correct answer = B. Patients with a history of renal insufficiency have a higher incidence of developing myalgias, myopathy, and rhabdomyolysis with use of HMG CoA reductase inhibitors (statins), especially with those that are renally eliminated as drug accumulation can occur. The other populations have not been reported to have a higher incidence of this adverse effect with HMG CoA reductase inhibitors.

An 80-year-old male is taking warfarin indefinitely for the prevention of deep venous thrombosis. He is a compliant patient with a stable INR and has no issues with bleeding or bruising. He is diagnosed with a urinary tract infection and is prescribed sulfamethoxazole/ trimethoprim. What effect will this have on his warfarin therapy? A. Sulfamethoxazole/trimethoprim will decrease the anticoagulant effect of warfarin. B. Sulfamethoxazole/trimethoprim will increase the anticoagulant effect of warfarin. C. Sulfamethoxazole/trimethoprim will activate platelet activity. D. Sulfamethoxazole/trimethoprim will not change anticoagulation status.

Correct answer = B. Sulfamethoxazole/trimethoprim has a significant drug interaction with warfarin, such that it will inhibit warfarin metabolism. Therefore, sulfamethoxazole/ trimethoprim will cause increased anticoagulation, and the patient will need to have his warfarin dose decreased and INR checked frequently while he is on this antibiotic.

Which of the following drugs specifically inhibits calcineurin in the activated T lymphocytes? A. Basiliximab. B. Tacrolimus. C. Prednisone. D. Sirolimus. E. Mycophenolate mofetil.

Correct answer = B. Tacrolimus binds to FKBP-12, which, in turn, inhibits calcineurin and interferes in the cascade of reactions that synthesize interleukin-2 (IL-2) and lead to T-lymphocyte proliferation. Although basiliximab also interferes with T-lymphocyte proliferation, it does so by binding to the CD25 site on the IL-2 receptor. Prednisone can affect not only T-cell proliferation but also that of B cells and is, therefore, nonspecific. Sirolimus, while also binding to FKBP-12, does not inhibit calcineurin. Mycophenolate mofetil exerts its immunosuppressive action by inhibiting inosine monophosphate dehydrogenase, thus depriving the cells of guanosine, a key component of nucleic acids.

CN is a 72-year-old male who is treated for hyperlipidemia with high-dose atorvastatin for the past 6 months. He also has a history of renal insufficiency. His most recent lipid panel shows an LDL cholesterol level of 131 mg/dL, triglycerides of 510 mg/dL, and HDL cholesterol of 32 mg/dL. His physician wishes to add an additional agent for his hyperlipidemia. Which of the following choices is the best option to address CN's dyslipidemia? A. Fenofibrate. B. Niacin. C. Colesevelam. D. Gemfibrozil. E. Ezetimibe.

Correct answer = B. This patient has significantly elevated triglycerides and low HDL. Niacin can lower triglycerides by 35% to 50% and also raise HDL levels. The fibrates (fenofibrate and gemfibrozil) should not be used due to CN's history of renal insufficiency. Use of colesevelam is contraindicated because triglycerides are greater than 400 mg/dL. Ezetimibe can further lower LDL cholesterol but has modest effects on triglycerides versus niacin.

Which best describes the action of ACE inhibitors on the failing heart? A. ACE inhibitors increase vascular resistance. B. ACE inhibitors decrease cardiac output. C. ACE inhibitors reduce preload. D. ACE inhibitors increase aldosterone.

Correct answer = C. ACE inhibitors decrease vascular resistance, decrease preload, decrease afterload, and increase cardiac output. In addition, ACE inhibitors blunt aldosterone release.

SC is a 75-year-old white male who has HF. He is seen in clinic today, reporting shortness of breath, increased pitting edema, and a 5-pound weight gain over the last 2 days. His current medication regimen includes losartan and metoprolol succinate. SC has no chest pain and is deemed stable for outpatient treatment. Which of the following is the best recommendation? A. Increase the dose of metoprolol succinate. B. Start hydrochlorothiazide. C. Start furosemide. D. Discontinue losartan.

Correct answer = C. As it is possible that SC is having a HF exacerbation, increasing the dose of the β-blocker is not indicated at this time. There is no reason to stop losartan, based on the information we have. Loop diuretics are preferred over thiazide diuretics when patients require diuresis immediately.

Which one of the following drugs binds bile acids in the intestine, thus preventing their return to the liver via the enterohepatic circulation? A. Niacin. B. Fenofibrate. C. Cholestyramine. D. Fluvastatin. E. Lovastatin.

Correct answer = C. Cholestyramine is an anion-exchange resin that binds negatively charged bile acids and bile salts in the small intestine. The resin/bile acid complex is excreted in the feces, thus preventing the bile acids from returning to the liver by the enterohepatic circulation. The other choices do not bind intestinal bile acids.

In which disease state is cilostazol contraindicated? A. Peripheral arterial disease. B. Gout. C. Heart failure with reduced ejection fraction. D. Osteoporosis.

Correct answer = C. Cilostazol is contraindicated in heart failure with reduced ejection fraction because it is a phosphodiesterase inhibitor and acts as a positive inotrope (which can lead to sudden cardiac death).

A 45-year-old male presented to the emergency department 18 hours after ingesting an unknown product. On presentation, he is tachycardic, hypertensive, tachypneic, and complaining of flank pain. A metabolic panel is obtained, and the patient has a large anion gap acidosis, an increased creatinine, and hypocalcemia. Which substance was most likely ingested? A. Methanol. B. Acetaminophen. C. Ethylene glycol. D. Iron. E. Opioids.

Correct answer = C. Ethylene glycol produces a metabolic acidosis from the toxic metabolites. The formation of calcium oxalate crystals, which can be found on urinalysis, leads to hypocalcemia and renal failure. The treatment regimen for this patient would include intravenous fomepizole, if some of the parent compound was still present, and hemodialysis. Thiamine and pyridoxine are the cofactors involved in the metabolism of ethylene glycol. Methanol may produce a metabolic acidosis as well, but its target organ of toxicity is the eyes instead of the kidneys as with ethylene glycol. Acetaminophen toxicity may produce upper quadrant pain within the first 24 hours, but vital sign abnormalities are not usually found during this time frame. Iron toxicity may also produce a metabolic acidosis and tachycardia. However, hypocalcemia does not occur. Opioid toxicity, as mentioned in Chapter 14, usually presents with CNS and respiratory depression, not tachycardia and hypertension.

Which must heparin bind to in order to exert its anticoagulant effect? A. GP IIb/IIIa receptor. B. Thrombin. C. Antithrombin III. D. von Willebrand factor.

Correct answer = C. Heparin binds to antithrombin III, causing a conformational change. This heparin/antithrombin III complex then inactivates thrombin and factor Xa.

Which is an appropriate choice for hypertension treatment during pregnancy? A. Aliskiren. B. Fosinopril. C. Hydralazine. D. Valsartan.

Correct answer = C. Hydralazine is an appropriate choice for a hypertensive pregnant patient. ACE inhibitors, ARBs, and the direct renin inhibitor, aliskiren, are all contraindicated in pregnancy due to their potential for fetal harm.

Which of the P2Y12 ADP receptor antagonists reversibly binds the receptor? A. Clopidogrel. B. Prasugrel. C. Ticagrelor. D. Ticlopidine.

Correct answer = C. Of the four P2Y12 ADP receptor antagonists, ticagrelor is the only one that reversibly binds the receptor. This is important when it comes to compliance. If a patient is not compliant, then the antiplatelet activity of ticagrelor stops when the drug is missed (since the platelets inhibited are not irreversibly inhibited as they would be with aspirin, clopidogrel, or prasugrel). On the other hand, the waiting period prior to surgery may be shorter in patients taking ticagrelor since it takes less time for the antiplatelet effect to wear off.

A 58-year-old female reports that she recently stopped taking her blood pressure medications because of swelling in her feet that began shortly after she started treatment. Which is most likely to cause peripheral edema? A. Atenolol. B. Clonidine. C. Felodipine. D. Hydralazine. E. Prazosin.

Correct answer = C. Peripheral edema is one of the most common side effects of calcium channel blockers. None of the other agents commonly cause peripheral edema.

What is the clinical term for angina caused by coronary vasospasm? A. Classic angina. B. Myocardial infarction. C. Prinzmetal angina. D. Unstable angina.

Correct answer = C. Prinzmetal angina is angina caused by vasospasm of the coronary arteries. It is also known as vasospastic or variant angina. The other answers refer to angina (with varying levels of severity) caused by atherosclerosis.

A 65-year-old male experiences uncontrolled angina attacks that limit his ability to do household chores. He is adherent to a maximized dose of β-blocker with a low heart rate and low blood pressure. He was unable to tolerate an increase in isosorbide mononitrate due to headache. Which is the most appropriate addition to his antianginal therapy? A. Amlodipine. B. Aspirin. C. Ranolazine. D. Verapamil.

Correct answer = C. Ranolazine is the best answer. The patient's blood pressure is low, so verapamil and amlodipine may drop blood pressure further. Verapamil may also decrease heart rate. Ranolazine can be used when other agents are maximized, especially when blood pressure is well controlled. The patient will need a baseline ECG and lab work to ensure safe use of this medication.

How is spironolactone beneficial in HF? A. Promotes potassium secretion. B. Agonizes aldosterone. C. Prevents cardiac hypertrophy. D. Decreases blood glucose.

Correct answer = C. Spironolactone antagonizes aldosterone, which in turn prevents salt/water retention, cardiac hypertrophy, and hypokalemia. Spironolactone has endocrine effects on hormones but not on glucose.

Which medication should be prescribed to all anginal patients to treat an acute attack? A. Isosorbide dinitrate. B. Nitroglycerin patch. C. Nitroglycerin sublingual tablet or spray. D. Ranolazine.

Correct answer = C. The other options will not provide prompt relief of angina and should not be used to treat an acute attack.

A 58-year-old woman is being treated for chronic suppression of a ventricular arrhythmia. After 1 week of therapy, she complains about feeling severe upset stomach and heartburn. Which antiarrhythmic drug is the likely cause of these symptoms? A. Amiodarone. B. Digoxin. C. Mexiletine. D. Propranolol. E. Quinidine.

Correct answer = C. The patient is exhibiting a classic adverse effect of mexiletine. None of the other agents listed are likely to cause dyspepsia.

All of the following medications can be useful for managing stable angina in a patient with coronary artery disease except: A. Amlodipine. B. Atenolol. C. Immediate-release nifedipine. D. Isosorbide dinitrate.

Correct answer = C. The short-acting dihydropyridine calcium channel blocker nifedipine should be avoided in CAD patients as this can worsen angina; however, the extendedrelease formulation can be used.

A 48-year-old hypertensive patient has been successfully treated with a thiazide diuretic for the last 5 years. Over the last 3 months, his diastolic pressure has steadily increased, and he was started on an additional antihypertensive agent. He complains of several instances of being unable to achieve an erection and not being able to complete three sets of tennis as he once did. Which is the likely second antihypertensive medication? A. Captopril. B. Losartan. C. Metoprolol. D. Minoxidil. E. Nifedipine.

Correct answer = C. The side effect profile of β-blockers, such as metoprolol, is characterized by interference with sexual performance and decreased exercise tolerance. None of the other drugs is likely to produce this combination of side effects.

A 45-year-old male who received a renal transplant 3 months previously and is being maintained on prednisone, cyclosporine, and mycophenolate mofetil is found to have increased creatinine levels and a kidney biopsy indicating severe rejection. Which of the following courses of therapy would be appropriate? A. Increased dose of prednisone. B. Hemodialysis. C. Treatment with rabbit antithymocyte globulin. D. Treatment with sirolimus. E. Treatment with azathioprine.

Correct answer = C. This patient is apparently undergoing an acute rejection of the kidney. The most effective treatment would be administration of an antibody. Increasing the dose of prednisone may have some effect but would not be enough to treat the rejection. Sirolimus is used prophylactically with cyclosporine to prevent renal rejection but is less effective when an episode is occurring. Furthermore, the combination of cyclosporine and sirolimus is more nephrotoxic than cyclosporine alone. Azathioprine has no benefit over mycophenolate.

A 34-year-old male with a history of a seizure disorder, maintained on phenytoin and phenobarbital, presented to the emergency department for CNS depression. The phenobarbital level was 70 mg/L (15 to 40 mg/L therapeutic range) and the phenytoin level was 15 mg/L (10 to 20 mg/L therapeutic range). He denies any acute ingestion. What therapy can be considered to enhance the elimination of phenobarbital without impacting the phenytoin? A. Multiple doses of activated charcoal. B. Gastric lavage. C. Urinary alkalinization. D. Whole bowel irrigation. E. Urinary acidification.

Correct answer = C. Urinary alkalinization enhances the elimination of the phenobarbital but does not affect the therapeutic phenytoin level. Sodium bicarbonate, 1 mEq/kg, is administered intravenously initially and then a sodium bicarbonate continuous infusion is titrated to maintain a urine pH of 7.5 to 8, without exceeding a serum pH of 7.55. Multiple doses of activated charcoal would lower the concentration of both medications, rendering the phenytoin subtherapeutic. Gastric lavage is a GI decontamination technique employed usually within the first hour after an acute ingestion of a life-threatening amount, to remove approximately 30% of the product in the stomach. Whole bowel irrigation is another GI decontamination modality involving administration of large quantities (up to 2 L/hour in adults) of a polyethylene glycol-balanced electrolyte solution via a nasogastric tube until the patient generates clear rectal effluent. Urinary acidification is no longer performed for substances such as amphetamines and quinidine.

A 62-year-old male taking warfarin for stroke prevention in atrial fibrillation presents to his primary care physician with an elevated INR of 10.5 without bleeding. He is instructed to hold his warfarin dose and given 2.5 mg of oral vitamin K1. When would the effects of vitamin K on the INR most likely be noted in this patient? A. 1 hour. B. 6 hours. C. 24 hours. D. 72 hours.

Correct answer = C. Vitamin K1 takes about 24 hours to see a reduction in the INR. This is due to the time required for the body to synthesize new coagulation factors.

What is the most common adverse effect associated with fixed-dose hydralazine/isosorbide dinitrate? A. Diarrhea. B. Drug-induced lupus. C. Headache. D. Heartburn.

Correct answer = C. While drug-induced lupus is a possibility with hydralazine, headache is the most common adverse effect.

A 60-year-old woman had a myocardial infarction. Which of the following should be used to prevent life-threatening arrhythmias that can occur post- myocardial infarction in this patient? A. Digoxin. B. Flecainide. C. Metoprolol. D. Procainamide. E. Quinidine.

Correct answer = C. β-Blockers such as metoprolol prevent arrhythmias that occur subsequent to a myocardial infarction. None of the other drugs has been shown to be effective in preventing postinfarct arrhythmias. Flecainide should be avoided in patients with structural heart disease.

A 41-year-old male pocket watch maker presents to the emergency department after he was found unconscious on the floor of the shop by a coworker. The coworker states that the patient complained of being cold this morning around 8 am (the central heat was broken, and the outdoor temperature was 34°F) and that since noon, he had been complaining of headache, drowsiness, confusion, and nausea. The clinician notices that he has cherry red skin. What is the most likely toxin causing his signs and symptoms? A. Ethylene glycol. B. Cyanide. C. Acetaminophen. D. Carbon monoxide. E. Methanol.

Correct answer = D. Although watch makers and other professionals who use electroplating may be at higher risk for cyanide exposure because many plating baths use cyanidecontaining ingredients (for example, potassium cyanide), this patient shows signs of carbon monoxide poisoning, such as cherry red skin, headache, confusion, nausea, and drowsiness leading to unconsciousness. The history also leads us to believe that this person may have been using a space heater to stay warm, which would be consistent with the description. A carboxyhemoglobin level should be obtained to confirm the exposure. Cyanide in low doses from such an occupational exposure can present with loss of consciousness, flushing, headache, and confusion. Chronically, workers may develop a rash after handling cyanide solutions. Also, an odor of bitter almonds may be present. An arterial blood gas and a venous blood gas could be obtained and compared to determine if cyanide is present (a lack of oxygen extraction would be present on the venous side). Ethylene glycol and methanol toxicity may cause alterations in mental status, but the history did not include anything suggesting a toxic alcohol ingestion. Acetaminophen toxicity is not consistent with this presentation.

A 50-year-old migrant worker comes to the emergency department from the field he was working in and complains of diarrhea, tearing, nausea and vomiting, and sweating. The clinician notices that he looks generally anxious and has fine fasciculations in the muscles of the upper chest as well as pinpoint pupils. Which antidote should he receive first? A. N-acetylcysteine. B. Sodium nitrite. C. Deferoxamine. D. Atropine. E. Fomepizole.

Correct answer = D. Atropine is appropriate for this patient, who has symptoms consistent with organophosphate (insecticide) poisoning. The mnemonic DUMBBELS (diarrhea, urination, miosis, bronchorrhea/bradycardia, emesis, lacrimation, salivation) can be used to remember the signs and symptoms of cholinergic toxicity. An anticholinergic antidote, atropine, controls these muscarinic symptoms, whereas the antidote pralidoxime treats the nicotinic symptoms like fasciculations (involuntary muscle quivering or twitching). N-acetylcysteine is the antidote for acetaminophen overdose and acts as a sulfhydryl donor. Sodium nitrite is one of the antidotes included in the old cyanide antidote kit (sodium nitrite and sodium thiosulfate). Deferoxamine is the chelating agent for iron. Fomepizole is the antidote for methanol and ethylene glycol.

Chemical warfare agents that had been manufactured in the 1950s were being stored at a military installation. Several civilian workers at the facility began to feel unwell, with symptoms that included dyspnea, abdominal cramps, and diarrhea. They also had copious nasal and tracheobronchial secretions. Which type of toxic compound is most likely to be the cause of these effects? (A) Aliphatic hydrocarbons (B) Botulinum toxins (C) Nitrogen mustards (D) Organophosphates (E) Rotenones

Highly potent organophosphate inhibitors of acetylcholinesterase (eg, sarin, tabun) have been developed for chemical warfare purposes. Their storage represents a potential toxicologic hazard. It is important to recognize the signs and symptoms of excess acetylcholine (DUMBBELSS; see Chapter 7), which include those described. The answer is D.

A 68-year-old man with a history of chronic heart failure goes on vacation and abandons his low-salt diet. Three days later, he develops severe shortness of breath and is admitted to the local hospital emergency department with significant pulmonary edema. The first-line drug of choice in most cases of acute decompensation in patients with chronic heart failure is (A) Atenolol (B) Captopril (C) Carvedilol (D) Digoxin (E) Diltiazem (F) Dobutamine (G) Enalapril (H) Furosemide (I) Metoprolol (J) Spironolactone

In both acute and chronic failure and systolic and diastolic heart failure, the initial treatment of choice is usually furosemide. The answer is H.

Which of the following drugs is most likely to increase this patient's triglyceride and VLDL cholesterol concentrations when used as monotherapy? (A) Atorvastatin (B) Cholestyramine (C) Ezetimibe (D) Gemfibrozil (E) Niacin

In some patients with familial combined hyperlipidemia and elevated VLDL, the resins increase VLDL and triglyceride concentrations even though they also lower LDL cholesterol. The answer is B.

An 81-year-old woman with type 2 diabetes presents to the emergency department in a coma and with tachypnea, tachycardia, hypotension, and severe lactic acidosis approximately 9 h after ingesting a number of her metformin tablets. Her serum glucose concentration is 148 mg/dL. Metformin is a base with a pKa of 12.4. The procedure that is most likely to improve her condition is (A) Administration of activated charcoal (B) Administration of glucagon (C) Administration of syrup of ipecac (D) Gastric lavage (E) Hemodialysis

In this woman with severe signs of poisoning due to the ingestion of metformin, hemodialysis can be used to accelerate the elimination of both metformin and lactic acid. Since most of the metformin has been absorbed by the time she presented (9 h after drug ingestion), efforts to decontaminate her gastrointestinal tract with activated charcoal, gastric lavage, or syrup of ipecac are unlikely to be beneficial. Furthermore, syrup of ipecac has fallen out of favor and should not be used in unconscious patients. Unlike other drugs used to treat type 2 diabetes, metformin in overdose is unlikely to cause hypoglycemia (see Chapter 41), and this patient's serum glucose is in the normal range so that glucagon administration is not required. The answer is E.

A 36-year-old man presents with swollen, painful heels, nail changes, and left lower back pain that wakes him from sleep. The back pain gets better with exercise. He reports 1-2 h of morning stiffness. He has a history of psoriasis and psoriatic arthritis since age 12 years. You decide to change his current regimen of indomethacin to a biologic that targets TNF-α. Which of the following is a chimeric monoclonal antibody that binds to TNF-α and inhibits its action? (A) Etanercept (B) Infliximab (C) Sirolimus (D) Trastuzumab (E) Thalidomide

Infliximab is a chimeric monoclonal antibody that binds to TNF-α. Etanercept also binds to TNF-α, but it is a chimeric protein containing a portion of the human TNF-α receptor linked to the Fc region of a human IgG. Thalidomide is a small molecule that appears to inhibit production of TNF-α. Trastuzumab is a humanized monoclonal antibody against HER-2/neu. The answer is B.

A patient with hypertension and angina is referred for treatment. Metoprolol and verapamil are among the drugs considered. Both metoprolol and verapamil are associated with which one of the following? (A) Diarrhea (B) Hypoglycemia (C) Increased PR interval (D) Tachycardia (E) Thyrotoxicosis

Neither β blockers nor calcium channel blockers cause diarrhea. Hypoglycemia is not a common effect of any of the antihypertensive drugs. Thyroid disorders are not associated with either drug group. However, calcium blockers, especially verapamil and diltiazem, and β blockers are associated with depression of calcium-dependent processes in the heart, for example, contractility, heart rate, and atrioventricular conduction. Therefore, bradycardia and increased PR interval may be expected. The dihydropyridines do not often cause cardiac depression, probably because they evoke increased sympathetic outflow as a result of their dominant vascular effects. The answer is C.

If the patient has a history of gout, which of the following drugs is most likely to exacerbate this condition? (A) Colestipol (B) Ezetimibe (C) Gemfibrozil (D) Niacin (E) Simvastatin

Niacin can exacerbate both hyperuricemia and glucose intolerance. The answer is D.

Which one of the following is characteristic of nifedipine treatment in patients with essential hypertension? (A) Competitively blocks angiotensin II at its receptor (B) Decreases calcium efflux from skeletal muscle (C) Decreases renin concentration in the blood (D) Decreases calcium influx into smooth muscle (E) Decreases calcium flux into the urine

Nifedipine is a prototype L-type calcium channel blocker and lowers blood pressure by reducing calcium influx into vascular smooth muscle. It has no effect on angiotensin-converting enzyme. Calcium efflux from skeletal muscle cells does not involve the L-type Ca channel. The plasma renin level may increase as a result of the compensatory response to reduced blood pressure. Calcium channel blockers have negligible effects on urine calcium. The answer is D.

A young woman employed as a dental laboratory technician complains of conjunctivitis, skin irritation, and hair loss. On examination, she has perforation of the nasal septum and a "milk and roses" complexion. These signs and symptoms are most likely due to (A) Acute mercury poisoning (B) Chronic inorganic arsenic poisoning (C) Chronic mercury poisoning (D) Excessive use of supplementary iron tablets (E) Lead poisoning

The "milk and roses" complexion, which results from vasodilation and anemia, is a characteristic of chronic inorganic arsenic poisoning, whereas patients with lead poisoning often have a gray pallor. Other signs and symptoms of arsenic poisoning include gastrointestinal distress, hyperpigmentation, and white lines on the nails. We hope you were not led astray by her employment. The answer is B.

A compound that is toxic to bone marrow cells in the early stages of development and that may also be leukemogenic is (A) Benzene (B) Carbon monoxide (C) Glyphosate (D) DDT (E) Pyrethrum

The aromatic hydrocarbon benzene is used as a solvent in industry. Long-term exposure is associated with increased risk of leukemia. The answer is A.

One year later, the patient returns complaining that his nitroglycerin works well when he takes it for an acute attack but that he is now having more frequent attacks and would like something to prevent them. Useful drugs for the prophylaxis of angina of effort include (A) Amyl nitrite (B) Esmolol (C) Sublingual isosorbide dinitrate (D) Sublingual nitroglycerin (E) Verapamil

The calcium channel blockers and the β blockers are generally effective in reducing the number of attacks of angina of effort, and most have durations of 4-8 h. Oral and transdermal nitrates have similar or longer durations. Amyl nitrite and the sublingual nitrates have short durations of action (a few minutes to 30 min). Esmolol (an intravenous β blocker) must be given intravenously and also has a very short duration of action. These drugs are of no value in prophylaxis. The answer is E.

A 5-year-old child was vomiting and was brought to the emergency department with sinus arrest and a ventricular rate of 35 bpm. An empty bottle of his uncle's digoxin was found where he was playing. Which of the following is the drug of choice in treating a severe overdose of digoxin? (A) Digoxin antibodies (B) Lidocaine infusion (C) Magnesium infusion (D) Phenytoin by mouth (E) Potassium by mouth

The drug of choice in severe, massive overdose with any cardiac glycoside is digoxin antibody, Digibind. The other drugs listed are used in moderate overdosage associated with increased automaticity. The answer is A.

After your patient has been receiving digoxin for 3 wk, he presents to the emergency department with an arrhythmia. Which one of the following is most likely to contribute to the arrhythmogenic effect of digoxin? (A) Increased parasympathetic discharge (B) Increased intracellular calcium (C) Decreased sympathetic discharge (D) Decreased intracellular ATP (E) Increased extracellular potassium

The effects of digitalis include increased vagal action on the heart (not arrhythmogenic) and increased intracellular calcium, including calcium overload, the most important cause of toxicity. Decreased sympathetic discharge and increased extracellular potassium and magnesium reduce digitalis arrhythmogenesis. The answer is B.

When nitrates are used in combination with other drugs for the treatment of angina, which one of the following combinations results in additive effects on the variable specified? (A) Beta blockers and nitrates on end-diastolic cardiac size (B) Beta blockers and nitrates on heart rate (C) Beta blockers and nitrates on venous tone (D) Calcium channel blockers and β blockers on cardiac force (E) Calcium channel blockers and nitrates on heart rate

The effects of β blockers (or calcium channel blockers) and nitrates on heart size, force, venous tone, and heart rate are opposite. The effects of β blockers and calcium channel blockers on the variables specified here are the same. The answer is D.

During the next week, the patient was started on warfarin and her enoxaparin was discontinued. Two months later, she returned after a severe nosebleed. Laboratory analysis revealed an INR (international normalized ratio) of 7.0 (INR value in such a warfarin-treated patient should be 2.0-3.0). To prevent severe hemorrhage, the warfarin should be discontinued and this patient should be treated immediately with which of the following? (A) Aminocaproic acid (B) Desmopressin (C) Factor VIII (D) Protamine (E) Vitamin K1

The elevated INR indicates excessive anticoagulation with a high risk of hemorrhage. Warfarin should be discontinued and vitamin K1 administered to accelerate formation of vitamin K-dependent factors. The answer is E

Six months after beginning atorvastatin, the patient's total and LDL cholesterol concentrations remained above normal, and he continued to have anginal attacks despite good adherence to his antianginal medications. His physician decided to add ezetimibe. Which of the following is the most accurate description of ezetimibe's mechanism of an action? (A) Decreased lipid synthesis in adipose tissue (B) Decreased secretion of VLDL by the liver (C) Decreased gastrointestinal absorption of cholesterol (D) Increased endocytosis of HDL by the liver (E) Increased lipid hydrolysis by lipoprotein lipase

The major recognized effect of ezetimibe is inhibition of absorption of cholesterol in the intestine. The answer is C.

A compound or group of compounds that damages the skin and whose use in manufacturing has largely been eliminated because of extensive persistence in the environment and bioaccumulation is (A) Aromatic hydrocarbons such as benzene (B) Dichlorvos (C) Phenoxyacetic acids such as 2,4-dichlorophenoxyacetic acid (D) Polychlorinated biphenyls (PCBs) (E) 2,3,7,8-tetrachlorodibenzo-p-dioxin (TCDD)

The polychlorinated biphenyls (PCBs) are dermatotoxic drugs that persist in the environment and accumulate in living organisms. PCBs have been banned from manufacture in the United States since 1979. However, many electrical transformers still retain traces of them. The answer is D.


संबंधित स्टडी सेट्स

Life insurance premiums, Proceeds and Beneficiaries

View Set

First Aid PHED 1306 EXAM 1 ALL QUESTIONS

View Set

ORGL 3332 Behavior, Ethics, Leadership II

View Set

introduction to business ch's 1,2,3

View Set

last F*king bus law test EVER!!!!!

View Set